Проезд на зеленый мигающий: ПДД РФ, 6. Сигналы светофора и регулировщика \ КонсультантПлюс

Содержание

Проезд на мигающий зеленый | AVTONAUKA.RU


Комментариев нет

2-я статья из 3 в серии Дорожные ситуации. Проезд на зеленый сигнал

Разбор дорожной ситуации: попытка проехать перекресток на мигающий зеленый сигнал светофора, фактически, на смене сигналов, привела к легкому ДТП. Кроме автомобиля исполнителя и дорожного знака никто не пострадал.

В соответствии с пунктом 6.2 ПДД зеленый мигающий сигнал светофора разрешает движение и предупреждает, что вскоре будет включен запрещающий сигнал.

Зеленый сигнал мигает примерно 3 секунды, и этого времени, обычно, бывает достаточно для того, чтобы заехать на регулируемый участок дороги (например, перекресток) или замедлить движение и остановиться перед светофором (перед стоп-линией).

Когда впереди по движению зеленый сигнал начинает мигать, то в зависимости от направления и характера своего движения (скорость, плотность потока, расстояние до перекрестка и др. ), вы примерно понимаете, успеете проехать или не успеете.

В разных ситуациях, впереди, на перекрестке, могут быть встречные машины, ожидающие свою возможность повернуть налево, но, как обычно бывает, желания других в расчет не принимаются, тем более, что пункт 13.4 ПДД обязывает их уступить дорогу тем, кто едет через перекресток «прямо» или поворачивает направо.

Встречных ТС, которым — налево, может и не быть, тогда без лишних сомнений можно «придавить» газ, и успеть проехать на зеленый мигающий сигнал до того, как он переключится на желтый.

Все, о чем было изложено выше, это, так сказать, правовая часть, т.е. правомерность проезда на мигающий зеленый сигнал светофора. Но кроме правовой стороны этой ситуации еще необходимо учесть дорожные условия, в которых «все это» будет происходить.

 

Управляемость автомобиля напрямую зависит от качества сцепления его колес с дорожным покрытием.

В теплое время года сцепление колес автомобиля с дорогой почти идеальное и управляемость авто хорошая, т. е. автомобиль ведет себя предсказуемо: если вы повернули руль, то он поворачивает, когда тормозите — он останавливается.

В холодное время года, когда на дороге снег, гололед, в общем, скользко, сцепление колес с дорогой ухудшается: чем выше скорость автомобиля, тем больше вероятность срыва колес в скольжение с последующей потерей управления автомобилем.

Соответственно, принимая решение проехать/не проехать или как сманеврировать, необходимо учитывать дорожные и погодные условия.

Посмотрите короткий видеоролик.

Впереди по движению регулируемый перекресток. Широкая дорога с разделительной полосой плотно покрыта укатанным снегом. Видеорегистратор «движется» в правом ряду и выступает в роли «свидетеля» в этой дорожной ситуации.

Начинает мигать зеленый сигнал, в кадре слева появляется белый седан, на приличной скорости стремящийся к перекрестку по левой полосе. Проезжая часть с его стороны свободна до перекрестка. На пересечении стоят несколько встречных машин, остановившихся на для поворота налево (выполняют требование пункта 13.4).

 

Если судить по реакции ожидающих водителей, то они понимали, что белый седан не станет останавливаться, а продолжит движение через перекресток (в итоге, тот заехал уже на желтый). Т.е. подъезжающий седан был хорошо виден всем, ну или почти всем.

В какой-то момент, пока мигал зеленый сигнал, один из встречных ожидающих начал движение, чем вынудил водителя белого седана попытаться его объехать, в целях избежать ДТП. И это ему даже удалось, после чего седан занесло на дорожный знак, с которым он и столкнулся.

Водитель седана не рассчитывал на то, что ему придется на большой скорости объезжать автомобиль по укатанному снегу. Он, ведь, только хотел успеть проехать «прямо». Но, как часто бывает, в людское желание вмешиваются обстоятельства, которые не принимаются в расчет.

Проехать перекресток помешал тот водитель кроссовера, который не уступил дорогу, т. е. тронулся с места, и вынудил водителя седана изменить траекторию движения. А погодные обстоятельства не позволили водителю седана безопасно сманеврировать на этом участке дороги (было скользко).

Кто более виноват в этом ДТП оставим решать компетентным лицам. Целесообразно ставить вопрос не кто виноват, а как избежать ДТП в этих или похожих обстоятельствах.

Когда вы подъезжаете к перекрестку или другому регулируемому участку на мигающий зеленый сигнал светофора, то нельзя однозначно утверждать, как лучше поступить: остановиться или ехать. В одних обстоятельствах, предположим, летом, можно успеть проехать, в других — есть смысл не спешить, например, как в нашем случае.

Ведь, вполне возможно, что если бы подобное произошло в теплое время года, то скорее всего, обошлось бы без ДТП — заноса не случилось бы. Но это всего лишь предположение.

На вопрос «как быть в этом случае» однозначного ответа не существует, правильность решения будет зависеть от того, адекватно вы понимаете сложившуюся впереди ситуацию или нет. И так — во всем остальном по ходу движения.

А вот, что однозначно следует сделать, это с наступлением холодов необходимо переключится на «зимний» стиль вождения. В этом есть не просто большой смысл, а именно необходимость.

О летней и зимней манерах езды можно прочесть в статье Пришли холода – меняем стиль вождения.

Будьте внимательны за рулем!

Навигация по серии статей<< Проезд на только что включившийся зеленыйНе дал выехать с перекрестка >>

Автор: Сергей Довженко

Если есть желание поделиться прочитанным, ниже кнопки на выбор. Жмем, не стесняемся.

когда можно на желтый — журнал За рулем

Почти все водители сталкиваются с такой ситуацией. Впереди регулируемый перекресток. Горит зеленый. И только ты собрался пересечь перекресток, как загорелся желтый. Что делать?

Вообще, при приближении к перекрестку желательно убедиться, что регулируется он только светофором. Дело в том, что регулировщик на перекрестке будет поглавнее работающего светофора. Если же регулировщика нет, то водители при проезде такого перекрестка должны руководствоваться сигналами светофора.

Да, напомню, что если на всех светофорах одновременно мигают желтые огни, значит перекресток нерегулируемый. На такой желтый можно спокойно ехать, выполняя при этом требования дорожных знаков и руководствуясь правилами проезда нерегулируемых перекрестков.

светофор

Для удобства водителей на светофорах применяются специальные табло, в которых указывается оставшееся в секундах время до переключения на другой сигнал. Чаще всего секундомер встроен в желтую секцию светофора.

Для удобства водителей на светофорах применяются специальные табло, в которых указывается оставшееся в секундах время до переключения на другой сигнал. Чаще всего секундомер встроен в желтую секцию светофора.

Материалы по теме

Черный список: за что могут лишить водительских прав

Пункт 6. 2 Правил дорожного движения устанавливает следующие режимы работы светофоров:

ЗЕЛЕНЫЙ СИГНАЛ — разрешает движение;

ЗЕЛЕНЫЙ МИГАЮЩИЙ СИГНАЛ — разрешает движение и предупреждает о скором переключении на запрещающий.

ЖЕЛТЫЙ СИГНАЛ — запрещает движение. Правда, есть исключение. Так, в пункте 6.14 Правил указано, что можно продолжить движение и на желтый сигнал светофора, если водитель не может остановиться перед стоп-линией (светофором или перед пересекаемой проезжей частью), не применяя мер экстренного торможения. Кроме всего прочего, этот сигнал предупреждает о предстоящей смене режимов. Как правило, с зеленого на красный.

КРАСНЫЙ СИГНАЛ, в том числе мигающий, запрещает движение. Сочетание красного и желтого сигналов запрещает движение и информирует о предстоящем включении зеленого сигнала.

светофор

Отсчет в секундах позволяет водителям своевременно оценить ситуацию на дороге и принять правильное решение.

Отсчет в секундах позволяет водителям своевременно оценить ситуацию на дороге и принять правильное решение.

Итак, если светофор оборудован специальным табло, на котором отсчитывается время до смены на запрещающий сигнал, мы прикидываем, успеем ли выехать на перекресток на разрешающий. При этом нужно учесть следующее. Правила движения запрещают выезжать на перекресток (в том числе и на зеленый), если образовался затор, который вынудит водителя остановиться, создав препятствие для движения транспортных средств в поперечном направлении.

А пункт 13.8 Правил говорит о том, что при включении зеленого водитель должен уступить дорогу тем транспортным средствам, которые завершают движение через перекресток, и пешеходам, не закончившим переход проезжей части данного направления.

светофор

Оговоркой в Правилах, которая позволяет проехать на запрещающий желтый, пользуются фактически все водители. Хотя и могут остановиться перед перекрестком без оттормаживания в пол.

Оговоркой в Правилах, которая позволяет проехать на запрещающий желтый, пользуются фактически все водители. Хотя и могут остановиться перед перекрестком без оттормаживания в пол.

Материалы по теме

Тихоход на дороге: когда можно обгонять?

Если же на перекрестке заторов нет, а зеленый перед самым носом переключился на желтый, водитель должен остановиться перед стоп-линией (светофором или перед пересекаемой проезжей частью). Но, как я уже говорил, есть спасительная для всех оговорка. Если вы не можете вовремя остановиться (не оттормаживаясь в пол), то Правила разрешают продолжить движение и на желтый. Причем нигде не сказано, как проверить, была ли у вас возможность остановиться. Именно поэтому большинство водителей полагают, что проехать на желтый вполне себе нормальное явление. А некоторые оправдывают свои действия тем, что следующий за ними водитель может поздно среагировать, и произойдет одна из самых распространенных аварий.

С точки зрения правоприменительной практики наказать водителя, проехавшего перекресток на желтый (даже если была возможность остановиться, но он ее проигнорировал), фактически нельзя.

светофор

В данном случае автомобили должны начинать движение только при включенной стрелке, разрешающей движение направо. Но некоторые водители уже настолько привыкли к «разрешенному» желтому, что умудряются проскакивать и на красный. А это уже чревато серьезными авариями.

В данном случае автомобили должны начинать движение только при включенной стрелке, разрешающей движение направо. Но некоторые водители уже настолько привыкли к «разрешенному» желтому, что умудряются проскакивать и на красный. А это уже чревато серьезными авариями.

Материалы по теме

BMW придумала приложение для ловли «зеленой волны»

С точки зрения безопасности такие действия могут повлечь весьма печальные последствия. Столкновения на перекрестках одни из самых опасных. Как правило, такие происшествия заканчиваются человеческими жертвами.

Тем не менее на большинстве перекрестков режим работы светофоров устроен таким образом, что пока горит желтый сигнал с вашего направления, на других горит красный. Тем самым всем водителям, «не успевающим вовремя затормозить на желтый», фактически дарят несколько лишних секунд на проезд перекрестка. Вот только такая вольность приводит к самоуверенности водителей, которые привыкают сначала проезжать на желтый, а потом их уже и красным сигналом светофора не остановить.

Проезд на желтый: можно или нет?

Почти все водители сталкиваются с такой ситуацией. Впереди регулируемый перекресток. Горит зеленый. И только ты собрался пересечь перекресток, как загорелся желтый. Что делать?

Проезд на желтый: можно или нет?

Фото: Сергей Смирнов и Петр Ковалев/ТАСС

Как лучше проезжать перекресток на «мигающий зеленый»

Иван-16. 05.2022

Хочу сказать спасибо этой компании за работу. Продавал свой Додж Гранд. Состояние машины было нормальное. И естественно не хотелось продешевить и хотелось быстро продать. Поэтому и решил обратиться к скупщикам авто. Быстро договорились о встрече и в течение 2-х часов машина была продана. Деньги были выплачены сразу после заключения договора. Большое спасибо за отличный сервис!

Вадим-04.05.2022

Решил продать свою Киа Рио 2001 года. Позвонил в вашу компанию. Приехали на следующий день. Ребята управились всего за час. С меня требовалось только согласие. Сергей и Дмитрий немного поторговались, обрисовали рыночную ценовую политику. Все честно и позитивно. Спасибо вам за оперативную помощь! Если что, обращаться буду только к вам. Удачи вам, вы молодцы!

Дмитрий-25.04.2022

Пришлось срочно продать автомобиль BMW 7 2007 г. выпуска. Представители фирмы «Автовыкуп» приехали в день обращения, согласовав со мной конкретное время. Автомобиль проверили оперативно, быстро заключили договор купли-продажи, сразу выдали приемлемую сумму наличными. Благодарю за оперативность! 

Ольга-18.04.2022

Срочно продавала Mazda 3, 2006 г. с большим пробегом. Долго искала перекупщиков через интернет. Случайно попалась на глаза ваша компания «Автовыкуп». Быстро пошли на контакт, машина продалась за 2 часа. Наличные получила на руки сразу же после заключения договора. Было приятно что Максим (директор фирмы) проявил личную заинтересованность в выкупе автомобиля по адекватной цене. Благодарю «Автовыкуп» за отличное обслуживание и желаю дальнейшего процветания вашей фирме!

Станислав-04.04.2022

Сервис супер! Для тех, кто хочет срочно продать машину, да еще и по справедливым расценкам. Приезжает грамотный оценщик. Ведет себя нормально, назначает хорошую цену. На рынке так выгодно точно не продать. Все сделали за пару часов. Не стал звонить, сразу поехал в фирму «Автовыкуп». Меня вежливо встретили. Через несколько минут подошел оценщик, осмотрел авто и через 15 минут назвал цену. Цена за Опель Астру выше средней неожиданно приятно удивила. Спасибо! Всем рекомендую!

Денис-30.03.2022

Когда продавал свой Ниссан Альмера Классик, звонил во все автовыкупы Минска. Эта фирма «Автовыкуп» предложила самую адекватную цену за авто. Обслуживание организовано на высоком уровне. Большое спасибо за качественную и быструю работу!

Никита-22.03.2022

Долго не мог продать свой Opel Astra 2007 года выпуска. Знакомый посоветовал обратиться в компании по автовыкупу. Обзвонил несколько фирм, и выбрал вашу. Выкупили машину по цене близкой к той, на которую я и рассчитывал. Деньги получил сразу. Удобно и быстро. Спасибо, буду рекомендовать вашу компанию своим друзьям и знакомым.

Николай-10.03.2022

Нужно было срочно продать мой автомобиль Volkswagen Polo, 2006 года выпуска. Рад, что обратился в эту компанию. Сделку оформили грамотно и оперативно. Не обманули. Цену выкупа назначили адекватную. Меня все устроило – и уровень обслуживания, и вежливое обращение.

Екатерина-17.02.2022

Нужно было срочно продать автомобиль Audi A5 (2001 г. в.). Обратилась в фирму «Автовыкуп». Сама продать не смогла бы, не было свободного времени. Позвонила в компании, занимающиеся срочным выкупом машин в Минске. Выбрала данную, поскольку устроила высокая сумма выкупа. Покупку оформили оперативно, деньги выдали сразу. Остались приятные впечатления о работе сотрудников фирмы, советую обращаться.

Андрей-05.02.2022

Пришло время продать старый автомобиль КИА Рио 2007 года выпуска и купить новую машину. Сам не стал заниматься продажей, быстрее это сделать с помощью интернета. Позвонил в несколько компаний, сделал выбор в пользу этой. Оценщик приехал вечером в день обращения, тогда же обо всем договорились и заключили сделку. Все получилось, спасибо!

Антон-12.01.2022

Обратился в эту компанию «Автовыкуп», чтобы выкупили мою битую Volvo S 30. Машину взяли в этот же день. В начале дня оставил заявку, договорились о встрече, обсудили тонкости оформления сделки. Вечером того же дня авто было продано за приемлемую сумму. Купля-продажа длилась не больше часа. Расплатились сразу, наличными. Доволен обслуживанием, благодарю!

Сергей- 08.12.2021

Надолго уезжаем жить за границу. Нужно было продать Mercedes s500, 2009 г. и Lexus RX 350, 2014 г. Обращались в несколько фирм по выкупу. Сумма выкупа устроила только в этой. О цене, которая нас устроила, заранее договорились по телефону. В назначенный нами день сотрудники фирмы приехали и выкупили наши машины. Профессионально сработано!

Егора-09.08.2021

Спасибо компании за срочный выкуп автомобиля Мазда 626 1999 г.в., несмотря на его плохое состояние. Действуют оперативно. Выкупили автомобиль через пару часов после того, как оставил заявку.

Вероники-09.06.2021

Услугами компании пользуюсь уже не первый раз. Некоторое время назад продала им свое б/у авто Ситроен Xsara. Теперь снова хочу обновить средство передвижения. В прошлый раз мне понравилась работа сотрудников компании. Поэтому решила обратиться снова. Продать авто получилось опять без проблем, сработали оперативно. Рекомендую всем! Огромное спасибо!!!

Владимира-17.06.2021

Выражаю компании искреннюю благодарность за срочный выкуп автомобиля БМВ 3-Series (E36) 2002 г. выпуска! Работу выполнили в кратчайшие сроки, быстро и качественно! Отдельное спасибо Маскиму и Дмитрию за оперативную работу.

Виктора-17.08.2021

Благодарен парням за профессионализм, быстрый отклик и дружелюбное отношение к клиентам. Помогли продать мой поломанный БМВ 2010 года выпуска. Ребята приехали в день обращения. Провели тщательный осмотр машины, сделали оценку. Расплатились сразу, всю сумму отдали наличными. Рекомендую!

Артема-20.08.2021

Спасибо компании «Автовыкуп» и лично Максиму за то, что сделали выходную оценку моему автомобилю Пежо 2000-го года, оперативно выкупили машину. Спасибо за профессионализм и вежливость!

Ирины-22.06.2021

Спасибо за срочный выкуп моего автомобиля Citroen 2014 года. Ребята откликнулись сразу, приехали через некоторое время после моего звонка. Осмотрели машину, проверили функции, назначили неплохую цену. После того, как оформили все документы, довезли меня до дома. Спасибо за качественное обслуживание и вежливое обращение!

ПДД 6.2 — Сигналы светофора

Что означает мигание зеленого сигнала светофора?

1.?Предупреждает о неисправности светофора.
2.?Разрешает движение и информирует о том, что вскоре будет включен запрещающий сигнал.
3.?Запрещает дальнейшее движение.

Для информирования водителей о предстоящей смене зеленого сигнала светофора на желтый Правилами предусмотрена возможность мигания зеленого сигнала. Этот сигнал разрешает движение ТС. Длительность мигания при этом должна составлять 3 сек.

Вы намерены повернуть налево. Ваши действия?

1.?Уступите дорогу трамваю.
2.?Проедете перекресток первым.
3.?Дождетесь разрешающего сигнала специального светофора и только после этого повернете.

Зеленый сигнал светофора разрешает Вам поворот налево. Движение трамвая на данном перекрестке регулируется светофором одноцветной сигнализации в виде буквы «Т» . Учитывая, что при таком сигнале движение трамваю запрещено, Вы можете проехать перекресток первым.

В каком случае Вы обязаны уступить дорогу трамваю?

1.?При движении прямо.
2.?В обоих перечисленных случаях.
3.?При повороте налево.

Включенный указатель поворота информирует вас о том, что пути движения вашего автомобиля и трамвая пересекаются на перекрестке. Учитывая, что данный сигнал светофора одновременно разрешает движение вам и водителю трамвая, вы обязаны уступить дорогу трамваю.

Вы намерены повернуть налево. Ваши действия?

1.?Повернете налево, остановитесь в разрыве разделительной полосы и, после включения разрешающего сигнала светофора на выходе с перекрестка, завершите маневр.
2.?Выполните маневр без остановки на перекрестке.

Зеленый сигнал светофора дает вам право двигаться налево. При этом вы должны выехать в намеченном направлении независимо от сигнала светофора на выезде с перекрестка.

Вы намерены повернуть налево. Ваши действия?

1.?Выехав на перекресток, остановитесь у стоп-линии и, дождавшись зеленого сигнала светофора на разделительной полосе, завершите маневр.
2.?Выполните маневр без остановки на перекрестке.

Зеленый сигнал светофора дает вам право двигаться налево. Но, поскольку на данном перекрестке перед светофором, установленным на разделительной полосе, имеется стоп-линия , Вы должны остановиться перед ней в ожидании зеленого сигнала.

Вы намерены проехать перекресток в прямом направлении. Ваши действия?

1.?Уступите дорогу трамваю.
2.?Проедете перекресток первым.

Включенный указатель поворота трамвая информирует о том, что пути движения вашего автомобиля и трамвая пересекаются на перекрестке. Учитывая, что зеленый сигнал светофора разрешает движение прямо, а трамвайный светофор в виде буквы «Т» запрещает трамваю поворот направо, вы можете проехать перекресток первым.

Вы намерены проехать перекресток в прямом направлении. Ваши действия?

1.?Проедете перекресток первым.
2.?Уступите дорогу только трамваю.
3.?Уступите дорогу трамваю и автомобилю.

Зеленый сигнал светофора дает право на движение вам и встречным ТС. При этом вы должны уступить дорогу только трамваю. Встречный легковой автомобиль проехать перекресток вместе с трамваем не имеет права.

Что означает мигание желтого сигнала светофора?

1.?Предупреждает о неисправности светофора.
2.?Разрешает движение и информирует о наличии нерегулируемого перекрестка или пешеходного перехода.
3.?Запрещает дальнейшее движение.

Желтый мигающий сигнал предупреждает о возможной опасности и применяется для обозначения нерегулируемого перекрестка или пешеходного перехода.

При повороте налево Вы:

1.?Должны уступить дорогу только легковому автомобилю.
2.?Имеете право проехать перекресток первым.
3.?Должны уступить дорогу обоим транспортным средствам.

Зеленый сигнал светофора дает право на движение всем ТС. Но, поворачивая налево, вы должны уступить дорогу трамваю, который имеет преимущество перед безрельсовыми ТС, а также встречному легковому автомобилю (ПДД 13. 4).

Красный мигающий сигнал или два попеременно мигающих красных сигнала светофора, установленного на железнодорожном переезде, означают:

1.?Движение разрешается с особой осторожностью.
2.?Движение запрещено.
3.?Светофорная сигнализация неисправна.

Красный мигающий сигнал или два попеременно мигающих красных сигнала светофора запрещают движение. Такая сигнализация применяется главным образом на железнодорожных переездах.

Вы намерены повернуть налево. Кому следует уступить дорогу?

1.?Только автомобилю.
2.?Только трамваю.
3.?Автомобилю и трамваю.
4.?Никому.

Зеленый сигнал светофора дает право на движение вам и встречным ТС. При этом, поворачивая налево, вы должны уступить дорогу трамваю и легковому автомобилю, поворачивающим направо.

Что означает сочетание красного и желтого сигналов светофора?

1.?Неисправна светофорная сигнализация.
2.?Вскоре будет включен зеленый сигнал.
3.?Вскоре будет включен красный сигнал.

С целью информирования участников движения о предстоящем включении зеленого сигнала светофора применяется сочетание красного и желтого сигналов. Такая комбинация сигналов запрещает движение.

В каком случае Вам не придется уступать дорогу другим участникам движения?

1.?Только если Вы намерены продолжить движение прямо.
2.?Только если Вы намерены продолжить движение прямо или направо.
3.?В любом случае.

Зеленый сигнал светофора разрешает движение вам и трамваю. При этом, если вы на данном перекрестке намерены двигаться налево или в обратном направлении, вам придется уступить дорогу трамваю, а при повороте направо — пешеходам. Таким образом, только при движении прямо никому уступать дорогу не требуется.

Вы намерены повернуть направо. Ваши действия?

1.?Проедете перекресток первым.
2.?Уступите дорогу обоим трамваям.
3.?Уступите дорогу только трамваю А.
4.?Уступите дорогу только трамваю Б.

Такой сигнал светофора дает право на движение Вам и водителям трамваев. А так как для поворота направо Вам необходимо пересечь трамвайные пути, Вы обязаны уступить дорогу обоим трамваям.

Штраф за проезд на желтый свет светофора (Статья 12.12 КоАП РФ)

О том, что уличное движение регулируется можно не упоминать. Если бы этого не было, то на наших дорогах царил бы полный хаос, что вполне логично. Такая логика совсем ни к чему, а значит соблюдение требований регулирования дорожного движения это все же неотъемлемая и взаимовыгодная практика для всех участников дорожного движения. Одним из основных элементов регулирования на наших дорогах является светофор. С его «рабочими» цветами и соответственно требованиями связанными с ними мы знакомы с самого детства. Красный – стой, желтый – приготовиться, а зеленый – путь свободен. На самом деле, когда мы подрастаем, правила игры становятся другими. Здесь для того чтобы мотивировать свои действия уже не достаточно сослаться на то, что когда-то об этом рассказывали нам родители… Каждый участник дорожного движения должен четко представлять, какие существуют требования и возможные вариации его поведения. Причем эти знания не должны ограничиваться словами исходя из точки зрения каждого лично заинтересованного, лучше упомянуть о федеральных законах, которые и станут определяющими в разрешении той или иной ситуации. Так скажем станут догмой, которая расставит все по местам.

Так вот, наше многозначительное вступление связано как раз с одной из таких ситуаций, когда автолюбитель проехал светофор на желтый цвет. Такое действие можно рассматривать двояко. Во-первых, как нарушение ПДД, так как цвет все же запрещающий, а второе как исключение. Именно обо всех этих альтернативах мы и расскажем в нашей статье.

Когда разрешён проезд на жёлтый сигнал светофора?

И начнём мы с того, что жёлтый свет, согласно ПДД, фактически является запрещающим. То есть, если он загорелся, вам нельзя продолжать движение через перекрёсток, пешеходный переход или другие участки дороги, перед которым установлен светофорный объект.

Формально такой сигнал выполняет 2 функции:

  1. жёлтый свет запрещает движение по ПДД,
  2. он также предупреждает о скорой смене сигнала на красный.

Такое положение дел диктует нам пункт 6.2 Правил дорожного движения. Вместе с тем, по официальной трактовке тут же содержится исключение в виде отсылки к другому пункту – 6.14, который предписывает возможность движения на жёлтый в том случае, если у водителя нет возможности остановиться в соответствии с требованиями ПДД, не применяя экстренное торможение.

А требования Правил к остановке на запрещающий сигнал светофора содержатся уже в пункте 6.13. Согласно ему, вы обязаны остановиться:

  • перед стоп-линией при её наличии,
  • если она отсутствует, то на перекрёстке перед пересекаемой проезжей частью, но не создавая помех движению пешеходов,
  • на иных участках дорог – перед самим светофором.

Таким образом, если у вас была возможность полностью остановиться на жёлтый сигнал светофора, но вы, тем не менее, проехали дальше, а на данном участке висит камера автофиксации нарушений, то штраф будет вполне законен. Вот только далеко не всегда… А, если быть точнее, то никогда.

Почему? Вы узнаете немного ниже.

Если это мигающий зелёный свет?

Нет, камера фотофиксации не может фиксировать проезд на мигающий зелёный, потому как он вообще не запрещает дальнейшее движение и носит лишь информирующий характер. Даже если вы едва успели проехать на разрешающий моргающий свет, то нарушения в любом случае не будет. Соответственно, и штраф с камеры за это будет незаконен.

Именно так это работает и на практике. Ещё не было случаев, чтобы водителям приходили «письма счастья» за проезд на мигающий зелёный. Разве что по ошибке.

Если речь идёт о движении на мигающий жёлтый?

Штрафуют ли камеры за мигающий жёлтый сигнал светофора в 2021 году? Вполне себе да. Но, опять же, речь идёт об ошибочной фиксации.

Дело в том, что сам по себе такой сигнал также не запрещает дальнейший проезд. Более того, он вообще делает перекрёсток нерегулируемым и становится таковым, когда, например, светофор неисправен либо на данном участке дороги организован такой режим движения, когда светофорный объект автоматически в определённое время (например, в ночное) переводится в моргающий режим. Чаще всего такое бывает на пешеходных переходах.

И вот если светофор сломался, вследствие чего загорелся жёлтый мигающий, то камера, висящая над ним, может продолжать фиксировать нарушения проезда. Хотя, это редко, потому как чаще всего средства автофиксации синхронизируются с режимом работы светофора, и при поломке последнего также перестают снимать.

Вас также заинтересует:

  • Штраф за неработающую подсветку заднего номера в вопросах и ответах
  • Есть ли штраф за езду с разбитым, лопнутым или треснувшим лобовым стеклом и какой?
  • Какой штраф за знак «Кирпич» на заправке, как его избежать и отменить?

Комментарии к пункту 6.14 ПДД

Достаточно неоднозначная формулировка пункта вызывает различного рода спорные ситуации, в которых выигрывает более осведомленная с юридическими тонкостями сторона. Так, собственник транспортного средства может ссылаться на то, что он успел проехать на желтый и при этом не мог затормозить, не создавая аварийную ситуацию.

Инспектор ГИБДД будет настаивать на том, что промежуточный сигнал все же является запрещающим. На этом основании сотрудник дорожной службы будет настаивать на привлечении автомобилиста к административной ответственности.

Фиксируют ли камеры проезд на жёлтый в 2021 году?

Да. Многие автолюбители встречали такое на практике на сегодняшний день. А сама такая возможность предусмотрена, разумеется, не только техническими возможностями самих средств автофиксации, но и законодательством. Так, ГОСТ Р57144-2016 в таблице №2 пункта 6.5 перечисляет нарушения, фиксируемые камерами ГИБДД, и среди них есть запрещающий сигнал (и данному событию назначено название «С2»).

В каком размере придёт штраф?

Выше мы писали, что сегодня предусмотрено 2 меры ответственности за проезд на запрещающий сигнал. Но сможет ли в 2021 году камера опознать повторное нарушение, и что будет грозить водителю в этом случае?

Да, камеры умеют распознавать повторность – если ранее в течение года вы привлекались по части 1 статьи 12.12 (без разницы, при проезде на камеру или остановил сотрудник ГИБДД), то придёт штраф 5 тысяч. Если быть точнее, то отсчитывается год не от предыдущего нарушения, а от исполнения – полной оплаты штрафа. И чаще всего поправка по повторности нарушения определяется сотрудником Центра автофиксации на месте при ручной проверке материалов с камеры (да, автофиксация – это не совсем автоматический режим).

Однако, если вас поймала за проездом на жёлтый камера, то лишения прав быть не может. Более подробно обо всём вышеописанном мы рассказывали в статье про повторность нарушений ПДД.

Я проехал на жёлтый – придёт ли мне штраф, как узнать?

К сожалению, доподлинно определить, зафиксировала ли камера конкретно ваш случай движения на жёлтый, невозможно. Вы об этом узнаете только при появлении соответствующего штрафа в одном из сервисов либо когда придёт письмо с копией постановления по почте.

Но в большинстве средств работает инфракрасный датчик, который можно увидеть при проезде через камеру. Особенно, в тёмное время суток видно, как она светится красным цветом.

Между тем, в ожидании санкции можно подготовиться для успешного её оспаривания. Ведь, как мы уже писали выше, фото- и видеофиксация проезда на жёлтый свет светофора вообще незаконна.

Что означает мигание

Светофор предназначен не только для подачи запрещающих и разрешающих движение сигналов. Согласно второму подпункту шестого пункта ПДД, мигающий промежуточный сигнал имеет свое значение.

Прерывистая подача желтого сигнала предупреждает участников движения о нерегулируемом участке проезжей части, а также информирует о вступлении в зону повышенной опасности. При обнаружении мигания промежуточного сигнала автомобилисты и пешеходы должны повысить бдительность.

Подобное привлечение внимание не означает немедленную остановку. Участники движения должны лишь снизить скоростной режим и подготовиться к пересечению участка повышенной опасности.

Как обжаловать штраф?

Рассказу о формальной подаче жалобы мы посвятили подробную статью о порядке оспаривания штрафа ГИБДД. Поэтому здесь отметим особо важные тонкости и нюансы касательно обсуждаемой нами темы.

  • Как мы уже упомянули выше, обжаловать штраф с камеры за жёлтый следует в любом случае. Да, мы не можем, конечно же, гарантировать 100-процентного удовлетворения жалобы. Однако, правильно изложив факты и отсылки на законодательство 2021 года, постановление должны отменить.
  • Куда подавать заявление, указано на оборотной стороне копии постановления – в нашем случае это региональный ЦАФАП.
  • Вам нужно успеть подать жалобу в течение 10 суток после получении копии постановления. Если письмо вам не приходило, то лучше всё равно направьте заявление сразу, как только узнаете о штрафе, так как нередки случаи, когда на почте будет стоять отметка о безуспешном вручении вам письма с постановлением, хотя, вы об этом и подозревать не могли.

А теперь, собственно, перейдём к обоснованности такого штрафа! То есть в жалобе нам нужно будет указать, на каком основании в ЦАФАП обязаны отменить штраф за проезд на жёлтый сигнал. И оснований таких может быть несколько.

Законен ли штраф за жёлтый?

Итак, начнём мы с того, что, согласно административному законодательству 2021 года, любые сомнения в вашей виновности трактуются в вашу пользу. Это означает, что если есть хотя бы малейшее сомнение в том, что вы виноваты в нарушении ПДД, то постановление должны отменить. И только когда вина будет на 100% доказана, штраф становится законным. Так диктует нам статья 1.5 КоАП о презумпции невиновности.

Однако, тут же содержится Примечание, что это не работает в случае с камерами видео- или фотофиксации нарушений. Да и штраф здесь выносится собственнику автомобиля. Тем не менее, общий принцип закона должен работать в общем и целом. А это один из главных принципов в «административке».

А теперь вернёмся к нашей теме и поставим вопрос следующим образом: сможет ли камера автофиксации определить, была ли возможность у водителя остановиться без экстренного торможения? Если установка не может чётко ответить на данный вопрос при фиксации, то здесь и нарушается общий принцип законодательства – любые сомнения в виновности лица трактуются в его пользу.

Зачем нам всё это? Вот зачем!

Камеры фотофиксации вообще нельзя настраивать на жёлтый по ГОСТу!

На практике 2021 года вовсе не предусмотрена возможность работы средств автоматической фиксации для выявления такого нарушения, как проезд на жёлтый сигнал. Именно по той причине, что в ПДД существует условие, когда такое движение не запрещено – невозможность остановиться без резкого торможения. Умеет ли камера определять такую возможность? Практически ни одна не умеет.

Тем не менее, подкрепим вышеуказанное утверждение официальной трактовкой. Тот же ГОСТ Р57144-2016, только уже в таблице №3 пункта 6.5 содержит условия для возможности фиксации различных нарушений средствами автофиксации.

И здесь содержатся следующие требования для обеспечения события «Ф3», являющегося фиксацией проезда на запрещающий сигнал светофора.

  • Должны быть обеспечены одна или несколько фотографий с расположением автомобиля до стоп-линии, а на изображении должен быть виден сигнал светофора.
  • Либо вариант, фотографий машины во время или после проезда перекрёстка, пешеходного перехода (если светофор регулирует его) или же во время проезда ж/д переезда после шлагбаума или светофора и до ближайшего рельса либо после него. И здесь также обязательно наличие в кадре запрещающего сигнала светофора.

Однако, ключ здесь кроется в том, что во всех перечисленных условиях для возможности автофиксации проезда на запрещающий сигнал указывается, что в кадре фотоматериалов, полученных средствами такой фиксации, должен быть именно красный свет, а не жёлтый.

Таким образом, если в отношении вас вынесено постановление со штрафом, и вы уверены (либо на фотоматериалах в копии документа видно), что проехали именно на жёлтый, а не на красный сигнал, то нужно обжаловать штраф. И основание можно указать примерно следующее:

В соответствии с пунктом 6.5.1 Государственного стандарта ГОСТ Р57144-2016, «Технические средства автоматической фотовидеофиксации должны обеспечивать фиксацию событий (в том числе административных правонарушений) и формирование материалов в соответствии с требованиями, приведенными в таблицах 1-3.»

В соответствии с обозначениями материалов событий Ф3, на фотоматериалах в таких технических средства должен фиксироваться запрещающий (красный) сигнал транспортного светофора для определённого направления движения.

На основании изложенного следует сделать правильный вывод о том, что технические средства автоматической фотовидеофиксации не применяются для фиксации нарушения проезда на запрещающий жёлтый сигнал светофора.

Кроме того, я двигался(двигалась) в соответствии с пунктом 6.14 Правил дорожного движения, не имея возможности остановиться в соответствии с требованиями пункта 6.13 на жёлтый сигнал.

Если по фото не видно сигнал светофора?

Как мы указали выше, на фотоматериалах, полученных при проезде перекрёстка, к примеру, на жёлтый светофор под камерой, должен быть виден текущий сигнал во время фиксации нарушения. Если этого нет, то камера работает не по ГОСТу, и это также влечёт законную отмену назначенного штрафа.

Одна фотография должна быть или две?

А вот это уже не имеет большого значения. Дело в том, что Госстандарт предусматривает и одну, и две и более фотоматериалов. А вообще, как правило, камеры снимают видеоматериал, просто вам по почте на бумаге видео ну никак не прикрепить, и потому отправляются просто отдельные кадры с записи в качестве обоснования назначенного штрафа.

Ещё кое-что полезное для Вас:

  • Можно ли заказать и законно ездить без штрафа с госномерами без флага России?
  • Какой размер штрафа за парковку и движение по тротуару? Обжалование и тонкости ПДД
  • Нужно ли соблюдать уменьшенные дорожные знаки по ПДД?

Если фото вовсе отсутствует в материалах?

Это также несущественный недостаток полученной вами копии постановления. Дело в том, что, хотя ГОСТ и предусматривает обязательность хотя бы одной фотографии, нигде не говорится о том, что она обязательно должна быть показана вам.

А при попытке обжалования постановления только на это основании инспектор ЦАФАП может направить в суд или рассматривающему дело должностному лицу имеющуюся фотографию. То есть отсутствие её в копии постановления вовсе не означает её отсутствие в принципе.

Можно ли обжаловать, если у меня не было возможности остановиться без торможения?

Можно, но только если это реально так и есть. Как мы уже описали выше, чаще всего полученный с камеры при проезде на жёлтый материал представляет собой видеозапись проезда. И, если суд, например, решит заморочиться (то есть рассмотреть дело, как того требует закон – всесторонне), то сможет определить по видео, была ли реально у вас возможность встать перед стоп-линией или в другом требуемом месте, не прибегая именно к экстренному торможению.

То есть в таком случае по видеоролику можно легко определить скорость вашего движения и фазу смены сигнала светофора с мигающего зелёного на жёлтый в соответствии с дальностью расположения вашей машины в этот момент от стоп-линии. По определённой скорости примерно понятно, требуется для остановки применять экстренное торможение или нет. Конечно, всё это рассматривающий жалобу будет определять не на глаз, а путём проведения экспертизы.

Верховный Суд разрешил проезд на жёлтый сигнал светофора

Марка

Модель

Оставьте ваши контактные данные:

По телефону

На почту

Уточните удобное время для звонка:

День/дата

  • День/дата
  • Сегодня
  • Завтра
  • 17
  • 18
  • 19
  • 20
  • 21
  • 22

Часы

  • 8
  • 9
  • 10
  • 11
  • 12
  • 13
  • 14
  • 15
  • 16
  • 17
  • 18
  • 19
  • 20

Минуты

  • 10
  • 20
  • 30
  • 40
  • 50

Отправляя заявку я предоставляю свое согласие на сбор и обработку предоставленных мною личных персональных данных в соответствии с Законом Украины «О защите персональных данных»

Оставьте ваши контактные данные:

Уточните удобное время для звонка:

День/дата

  • День/дата
  • Сегодня
  • Завтра
  • 17
  • 18
  • 19
  • 20
  • 21
  • 22

Часы

  • 8
  • 9
  • 10
  • 11
  • 12
  • 13
  • 14
  • 15
  • 16
  • 17
  • 18
  • 19
  • 20

Минуты

  • 10
  • 20
  • 30
  • 40
  • 50

Прямо сейчас

Отправляя заявку я предоставляю свое согласие на сбор и обработку предоставленных мною личных персональных данных в соответствии с Законом Украины «О защите персональных данных»

Оставьте ваши контактные данные:

Выберите машину:

Марка

  • Сначала выберите дилера

Модель

  • Сначала выберите марку

Отправляя заявку я предоставляю свое согласие на сбор и обработку предоставленных мною личных персональных данных в соответствии с Законом Украины «О защите персональных данных»

Sample Text

Оставьте ваши контактные данные:

Выберите машину:

Марка

  • Сначала выберите дилера

Модель

  • Сначала выберите марку

Уточните удобное время для тест-драйва:

День/дата

  • День/дата
  • Сегодня
  • Завтра
  • 17 сентября
  • 18 сентября
  • 19 сентября
  • 20 сентября
  • 21 сентября
  • 22 сентября
  • 23 сентября
  • 24 сентября
  • 25 сентября
  • 26 сентября
  • 27 сентября
  • 28 сентября
  • 29 сентября

Часы

  • 8
  • 9
  • 10
  • 11
  • 12
  • 13
  • 14
  • 15
  • 16
  • 17
  • 18
  • 19
  • 20

Минуты

  • 00
  • 10
  • 20
  • 30
  • 40
  • 50

Отправляя заявку я предоставляю свое согласие на сбор и обработку предоставленных мною личных персональных данных в соответствии с Законом Украины «О защите персональных данных»

X

Оберіть мовну версію сайту. За замовчуванням autocentre.ua відображається українською мовою.

Слава Україні! Героям слава!
Ви будете перенаправлені на українську версію сайту через 10 секунд

ВРЕМЯ СИГНАЛА НА ШНУРЕ

Полезно определить временные параметры сигнала светофора как принадлежащие к одной из двух категорий: временные параметры местного перекрестка и временные параметры скоординированной работы. Минимум фазы или время изменения желтого цвета являются примерами параметров времени локального пересечения. Длина цикла пересечения и смещение являются примерами временных параметров координации. Временных параметров локального пересечения гораздо больше, чем временных параметров координации. Фактически, большинство систем определяют только три временных параметра координации: длину цикла, смещение и разделение. В следующем разделе этого отчета описываются методы, которые можно использовать для установки параметров времени локального пересечения, а в следующем разделе рассматриваются параметры скоординированной операции.

Основные временные параметры практически одинаковы для всех управляемых контроллеров. Существуют тонкие различия между различными программными реализациями; например, контроллеры NEMA определяют функцию принудительного отключения как функцию «на кольцо», в то время как другие реализации определяют функцию принудительного отключения как функцию «на фазу». Это различие не имеет большого значения для инженера по дорожному движению, который отвечает за разработку новых планов светофоров. Эти различия, однако, очень важны, когда результаты процесса оптимизации синхронизации сигналов реализуются в конкретном контроллере. Многие новые контроллеры автоматически рассчитывают настройку принудительного отключения, как только инженер определяет разделение.

Поскольку в большинстве юрисдикций используются контроллеры NEMA TS-1 и TS-2 или модели 170, основное внимание уделяется контроллерам с синхронизацией. Однако важно понимать, что во многих юрисдикциях используются контроллеры с предварительной синхронизацией. Большинство принципов, отмеченных в этом разделе, могут быть непосредственно применены к контроллерам с предварительной синхронизацией.

Чтобы узнать о текущем статусе синхронизации сигналов, читатель может обратиться к Практики и процедуры синхронизации сигналов: состояние практики (элемент IR-116), опубликованный Институтом инженеров транспорта (ITE). В этом отчете отражено текущее положение дел в области синхронизации сигналов светофора. Он был подготовлен в рамках продолжающейся программы, разработанной FHWA и ITE, чтобы уделять повышенное внимание качеству синхронизации сигналов светофора в США.

Принципы синхронизации контроллера с приводом

Некоторые из основных принципов синхронизации зеленого интервала в блоке контроллера с приводом от дорожного движения следующие:

Каждая фаза должна иметь минимальное время зеленого сигнала, чтобы у остановившегося транспортного средства, получившего зеленый сигнал, было достаточно времени, чтобы тронуться с места и частично пересечь перекресток до появления желтого сигнала. Это время иногда называют начальной частью зеленого интервала.

Каждому транспортному средству требуется достаточное количество зеленого времени, чтобы проехать от детектора до перекрестка. Это называется временем прохождения, удлинением транспортного средства или зазором. Промежуток относится к расстоянию между транспортными средствами, а также времени между транспортными средствами. Таким образом, каждое последующее срабатывание транспортного средства увеличивает время зеленого цвета фазы. При отсутствии встречных вызовов контроллер отдыхает. Расширения по-прежнему рассчитаны по времени, но не влияют на зеленый интервал.

Должно быть максимальное время, на которое зеленый интервал может быть продлен, если встречные машины ждут; это называется максимальным зеленым или пределом расширения.

После получения срабатывания от противоположной фазы запускается таймер максимального зеленого. Текущая фаза будет удерживать зеленый цвет до тех пор, пока время между активациями не превысит предустановленное расширение блока или промежуток. При обнаружении разрыва начнется желтый интервал замены, и контроллер перейдет к следующей фазе в соответствии с запросом. Это называется прекращением разрывом или разрывом.

Активация от другой фазы, полученная в течение любой части зеленого интервала, также запускает схему синхронизации максимального зеленого. Это также называется пределом расширения. Даже если срабатывания достаточно близки по времени, чтобы предотвратить завершение промежутка, максимальный предел прекратит зеленый интервал, когда истечет заданный максимум. Это называется завершением по максимальному зеленому цвету или max-out.

Основные настройки фазы активации

Все контроллеры с активацией поддерживают эти пять основных параметров синхронизации фазы: минимум зеленого, расширение, максимум зеленого, желтого и красного.

Минимальный зеленый (начальный)

Минимальный зеленый цвет — это первая рассчитанная по времени часть зеленого интервала. Продолжительность минимального зеленого цвета обычно зависит от количества транспортных средств, которые могут стоять в очереди между входным фазовым детектором и стоп-линией. Как правило, продолжительность фазы не должна быть меньше некоторого абсолютного минимума, например, от 5 до 10 секунд. Если пешеходы могут переходить дорогу с этой фазой, время их перехода также должно быть учтено и включено в минимальную продолжительность фазы.

Термин «минимум зеленый» на некоторых контроллерах называется «минимальный начальный». Есть тонкая разница. Минимальный зеленый цвет определяет продолжительность зеленого цвета, когда нет расширений. Когда контроллер использует минимальное начальное значение, минимальная продолжительность зеленого цвета равна сумме минимального начального значения и одного продления.

Минимальное время зеленого цвета — это минимально гарантированный зеленый цвет, который будет отображаться. Он установлен для того, чтобы транспортные средства, остановившиеся между датчиком на подъезде и стоп-линией, могли тронуться с места и выехать на перекресток. Следовательно, время этого интервала зависит от местоположения детектора и количества транспортных средств, которые могут храниться между детектором и стоп-линией.

Необходимо также учитывать время пешеходов. При отсутствии условий для пешеходов (указатели или кнопки) минимальный гарантированный зеленый цвет должен быть равен минимальному времени для пешеходов (прогулка + расстояние для пешеходов).

Один из методов, который можно использовать для расчета минимального количества гринов:

Минимум зеленый = 5 + 2n

Где:

«n» — количество транспортных средств, которое может храниться между стоп-линией и дальним датчиком в одной полосе. Это определяется путем деления расстояния (в футах) между стоп-линией и детектором на 25, поскольку 25 — это средняя длина транспортного средства плюс расстояние до остановки в футах.

При использовании определения стоп-линии минимальная зеленая полоса может быть довольно короткой, скажем, от 5 до 10 секунд. Эти очень короткие минимумы можно использовать на участках, где нет движения пешеходов, и на участках, где есть активированные пешеходные сигналы. Эти чрезвычайно короткие минимумы никогда не должны использоваться там, где есть вероятность перехода пешеходов с зеленым дисплеем.

Расширение (проход)

Параметр «Продление» (проезд) увеличивает зеленый интервал для каждого срабатывания автомобиля до максимального зеленого. Он начинает отсчет времени, когда транспортное средство снимается; то есть, когда транспортное средство проезжает над детектором. Этот период продления может быть прекращен таймером максимального продления или принудительным отключением.

Параметр фактического времени прохождения (удлинение транспортного средства или время промежутка) — это время, на которое будет продлеваться фаза для каждого срабатывания. Эта настройка представляет собой количество секунд, необходимое транспортному средству, движущемуся со скоростью приближения, для проезда от детектора до стоп-линии. Время прохождения служит двум целям: (1) это время прохождения от детектора до стоп-линии, и (2) это допустимый промежуток времени между активациями, при котором грин остается на этом подходе. Пока обнаружение транспортных средств происходит с более короткими интервалами, чем время проезда (допустимый промежуток), зеленый цвет будет сохраняться на этой фазе до тех пор, пока не истечет максимум или он не будет принудительно выключен.

Если интервал проезда слишком короткий, это может привести к быстрой остановке, а также прекращению действия грина до того, как движение транспортных средств будет надлежащим образом обслужено. Если установлен слишком длинный интервал проезда, это приведет к чрезмерным задержкам, а также к проблемам с безопасностью из-за неправильно рассчитанных интервалов между последними автомобилями.

Время прохождения рассчитывается следующим образом:

Время прохождения = Д/С

Где:

D — расстояние от стоп-линии до детектора в футах

S — скорость на подходе в футах в секунду.

Максимум Зеленый

Этот параметр времени определяет максимальное время, в течение которого фаза может быть зеленой при наличии конфликтующего вызова. Если конфликтующего вызова нет, он будет сбрасываться до тех пор, пока не произойдет встречный вызов.

Таймер максимального зеленого обычно блокируется во время скоординированной работы, когда максимальное время фазы определяется принудительным отключением фазы. Для большинства активированных фаз максимальное количество зеленого цвета следует рассматривать как ограничение безопасности, что-то, что заставляет фазу завершиться в условиях непрерывного спроса. Не существует общепринятой практики определения оптимальной настройки. Некоторые инженеры считают, что все фазы должны иметь одинаковые ограничения, такие как: 120 секунд для основных сквозных фаз, 90 секунд для второстепенных сквозных фаз и 60 секунд для фаз левого поворота. Другие устанавливают фазовые максимумы, пропорциональные спросу в критический период. Например, можно рассчитать разделение фаз с помощью метода критического движения, а затем установить максимальное значение зеленого цвета на 50 процентов выше, чем разделение фаз.

Желтый

Желтый интервал следует за зеленым интервалом в конце каждой фазы. Желтый интервал также называется интервалом «изменения» и определяет продолжительность желтого дисплея для этой фазы. Интервал изменения фазы сообщает водителям, что их фаза истекла, и они должны: (1) сделать безопасную остановку до стоп-линии или (2) проехать перекресток, если они находятся слишком близко к перекрестку, чтобы остановиться.

Следующее уравнение обычно используется для определения надлежащего интервала замены:

Желтое время = t + S / (2a +-64,4 g)

Где:

t — время восприятия/реакции водителя в секундах (обычно 1,0 секунды).

S скорость на подходе в футах в секунду.

a — скорость замедления в футах в секунду (обычно принимается как 10 футов в секунду в квадрате).

g = оценка приближения, процент оценки, деленный на 100 (добавьте для повышения и вычтите для понижения).

Красный

Красный интервал очистки (также известный как полностью красный интервал) следует за желтым интервалом каждой фазы. Он должен истечь, прежде чем можно будет начать следующую фазу последовательности. Обычно это одна-две секунды, но при заходах на посадку с более низкой скоростью нет ничего необычного в использовании очень короткой продолжительности от 0,0 до 0,5 секунд, поскольку желтое время от 3,0 до 4,0 секунд обеспечивает достаточно времени для выполнения требований как по изменению, так и по разрешению. .

Время красного = (W + L) / S

Где:

W — ширина пересечения в футах.
L — длина транспортного средства в футах (обычно принимается за 20 футов).
S — скорость на подходе в футах в секунду.

Параметры пешеходов

Существует два параметра времени для пешеходов: Walk и Flashing Don’t Walk (просвет для пешеходов). Там, где движение пешеходов происходит регулярно, пешеходам должно быть предоставлено достаточно времени, чтобы пересечь проезжую часть (MUTCD, раздел 4D. 03). Другими словами, если движение пешеходов не активировано, время, необходимое пешеходам для безопасного перехода дороги (сумма времени ходьбы и запрета ходьбы), будет определять минимальное время зеленого цвета для этой фазы. Когда предусмотрена кнопка нажатия пешеходом (приведение в действие), то при отсутствии нажатия пешеходом будут использоваться обычные минимальные значения для транспортных средств. Обратите внимание, что можно разработать сигнальный дизайн, в котором используется обнаружение пешеходов с помощью кнопок без использования дисплеев «Пешеход» и «Не иди».

Прогулка

Параметр Walk определяет продолжительность отображения сигнала ходьбы. В нормальных условиях интервал ходьбы составляет от 4 до 7 секунд. Это позволяет пешеходам иметь достаточную возможность покинуть бордюр до того, как будет показан интервал безопасного проезда. В особых случаях, например, на школьном перекрестке с большим количеством пешеходов, время ходьбы может превышать 7 секунд. Исследования показали, что очередь пешеходов из 24 и более человек может быть размещена за 7 секунд.

Мигает Не ходи

Параметр расстояния до пешеходов определяет продолжительность отображения мигающего сигнала «Не иди». Это время, необходимое пешеходу на пешеходном переходе, чтобы покинуть бордюр и выехать за пределы проезжей части (MUTCD, раздел 4E.02), прежде чем встречные транспортные средства получат зеленую индикацию.

Мигание «Не ходи» (пропуск для пешеходов) рассчитывается следующим образом:

Мигает «Не ходи» = W/S P

Где:

W — расстояние ходьбы (перехода) от бордюра до бордюра в футах.

S P — средняя скорость ходьбы в футах в секунду (обычно от 3,5 до 4 футов в секунду).

Настройки фазы объем-плотность

В 1950-х годах компания Automatic Signal представила два новых и сложных контроллера сигналов светофора, модель 1022 и модель 1033. Эти контроллеры имели гораздо больше схем синхронизации, чем любые контроллеры, использовавшиеся до или после. Хотя все временные параметры были хорошо обоснованы в теории; на практике многие фактически не улучшили работу перекрестка. Для этого было несколько причин: невозможно было определить, какой из множества таймеров фактически вызвал завершение определенной фазы; технология аналоговых схем синхронизации, доступная в то время, подвергалась изменениям в зависимости от температуры и влажности; и, как и сегодня, у немногих инженеров было время, необходимое для тонкой настройки операции. Из множества функций этих инновационных контроллеров две функции (переменное начальное значение и уменьшение зазора) были усовершенствованы и обычно доступны в современных контроллерах. Эти две особенности объясняются ниже.

Начальная переменная

Чтобы успешно использовать функции объемной плотности контроллера, необходимо иметь определение фазы выше по течению, чем обычно. Это позволяет контроллеру пробовать трафик на подходе с минимальным влиянием стоячей очереди. Обнаружение на расстоянии от 300 до 400 футов вверх по течению от стоп-линии является типичным при работе с объемной плотностью. При обнаружении так далеко, минимальный зеленый будет довольно длинным — 35 секунд с детектором на расстоянии 400 футов. Поскольку минимальное значение зеленого цвета такой продолжительности приведет к очень вялой работе при слабом спросе, для смягчения этих последствий используются две другие схемы синхронизации: добавленное начальное значение и максимальное начальное значение.

Добавлено начальное значение — этот интервал совпадает с минимальным зеленым интервалом. Значение представляет собой количество времени, которое добавляется при получении каждого срабатывания транспортного средства в течение начального периода. Фактическое минимальное время зеленого цвета, используемое контроллером, больше минимального зеленого времени или добавленной начальной суммы. Добавленный начальный уровень не может превышать максимальный начальный. Добавленный начальный параметр позволяет варьировать продолжительность минимума между фактическим минимумом и временем, необходимым для разрядки максимальной очереди, которая может быть сохранена между стоп-линией и детектором.

Максимальное начальное значение — это максимальный период времени, на который добавленное начальное значение может продлить начальный зеленый период. Максимальный начальный не может быть меньше минимального зеленого.

Уменьшение зазора

Уменьшение зазора — еще одна устаревшая концепция, используемая в современном контроллере. Этот параметр используется, чтобы позволить логике «поиска промежутка» активированного контроллера идентифицировать меньший промежуток в трафике, чтобы оправдать завершение фазы раньше, чем это подразумевается временем прохождения. Сокращение промежутка — это средство сокращения времени проезда или промежутка на основе времени, в течение которого встречные транспортные средства находились в ожидании. По сути, это приносит пользу ожидающим транспортным средствам, сокращая время, допустимое между транспортными средствами, прибывающими в зеленую фазу, прежде чем эта фаза будет завершена. С функцией уменьшения зазора связаны три временных параметра: время до сокращения, время до уменьшения и минимальный зазор.

Time-Before-Reduction — Этот период начинается, когда фаза становится зеленой и на конфликтующей фазе имеется обслуживаемый вызов. По завершении этого периода начинается линейное сокращение времени прохождения.

Time-To-Reduce — Этот период начинается, когда заканчивается Time-Before-Reduction, и управляет линейной скоростью уменьшения до тех пор, пока Gap не уменьшится до минимального Gap.

Минимальный зазор —Как и время проезда, этот параметр расширяет зеленый интервал на минимальное время промежутка для каждого срабатывания транспортного средства до максимального зеленого. Он начинает отсчет времени, когда привод транспортного средства удаляется. Этот период продления может быть прекращен максимальным зеленым или принудительным отключением.

Сводка параметров синхронизации контроллера

Обычно используются 12 параметров синхронизации контроллера:

  1. ХОДЬБА — устанавливает длину интервала ХОДЬБА.

  2. ДОСТУП ДЛЯ ПЕДАГ — устанавливает продолжительность мигающего интервала «НЕ ХОДИТЕ».

  3. MINIMUM GREEN — устанавливает длину начального состояния зеленого интервала.

  4. ВРЕМЯ ПРОЕЗДА — устанавливает приращение продления времени преимущественного проезда (зеленый) для каждого срабатывания транспортного средства в течение зеленого интервала.

  5. MAXIMUM GREEN — устанавливает максимальный предел, до которого зеленый интервал может быть расширен на фазе при наличии обслуживаемого запроса на конфликтующей фазе.

  6. ДОБАВЛЕНО НАЧАЛЬНОЕ — Функция плотности. Устанавливает количество секунд, на которое каждое транспортное средство (приведение в действие) увеличивает начальное состояние зеленого цвета в незеленое время на фазе 9.0003

  7. MAXIMUM INITIAL — Функция плотности. Устанавливает максимальный предел, до которого может быть увеличен начальный интервал на фазе.

  8. СНИЖЕНИЕ ВРЕМЕНИ B4 — Функция плотности. Устанавливает предустановленное время до того, как разрешенный промежуток (время прохода) начнет уменьшаться.

  9. ВРЕМЯ УМЕНЬШИТЬ—Функция плотности. Устанавливает время, в течение которого разрешенный зазор уменьшается от времени прохода до минимального зазора после того, как время до истечения времени до истечения времени сокращения.

  10. МИНИМАЛЬНЫЙ ЗАЗОР — Функция плотности. Устанавливает минимальное значение, до которого может быть уменьшен допустимый разрыв между срабатываниями на фазе с зеленым цветом по истечении времени уменьшения.

  11. ИЗМЕНЕНИЕ ЖЕЛТОГО — устанавливает длину интервала желтого цвета, следующего за интервалом зеленого.

  12. КРАСНЫЙ ЗАЗОР — устанавливает длину красного зазора, следующего за желтым интервалом.

FHWA — MUTCD — Издание 2003 г. Редакция 1 Глава 4K

Назад к 2003 г. Ред. 1 Таблица содержания  | Назад к части 4 Содержание

Раздел 4K.01 Общие Конструкция и работа проблесковых маячков

Поддержка:
Проблесковый маячок — это сигнал дорожного движения с одним или несколькими сигналами. разделы, работающие в режиме мигания. Он может обеспечить трафик контроль при использовании в качестве маяка управления перекрестком или предупреждения в альтернативное использование.

Стандарт:
Блоки проблесковых маячков и их крепления должны соответствовать положениям Главы 4D, за исключением случаев, указанных в настоящем документе.

Маяки должны мигать с частотой не менее не более 50 и не более 60 раз в минуту. Светлый период каждой вспышки должно быть не менее половины и не более две трети всего цикла.

Руководство:
Если используется в качестве дополнения к предупреждающему или регулирующему знаку, край корпус радиомаяка, как правило, должен быть расположен не ближе, чем 300 мм (12 дюймов) за пределами ближайшего края знака.

Опция:
Для уменьшения яркости можно использовать автоматическое затемняющее устройство. мигания желтой сигнальной индикации при работе в ночное время.

Участок 4K.02 Перекресток Контрольный маяк

Стандарт:
Контрольный маяк перекрестка должен состоять из одного или нескольких сигнальных лица, направленные в сторону каждого подхода к перекрестку. Каждый сигнал лицо должно состоять из одной или нескольких сигнальных секций стандартного лицо светофора с мигающим КРУГЛЫМ ЖЕЛТЫМ или КРУГОВЫМ КРАСНЫМ сигнальные индикаторы на каждом сигнальном лице. Они должны быть установлены и используется только на перекрестке для управления двумя или более направлениями путешествия.

Применение сигнала маяка управления перекрестком Индикация должна быть ограничена следующим:

  1. Желтый на одном маршруте (обычно главная улица) и красный на остальные подходы; и
  2. Красный для всех подходов (если ордер на многоходовую остановку доволен). Мигающие желтые сигнальные индикаторы не должны столкнуться с конфликтующими автомобильными подходами.

Знак СТОП должен использоваться на подходах к которым мигающая красная сигнальная индикация отображается на перекрестке Маяк (см. Раздел 2B.04).

Руководство:
Маяк контроля пересечения не должен быть установлен на постаменте. на проезжей части, если пьедестал не находится в пределах полосы движения или пешеходный остров.

Опция:
Индикация дополнительных сигналов может использоваться на одном или нескольких заходах на посадку. чтобы обеспечить достаточную видимость для приближающихся участников дорожного движения.

Контрольные маяки на перекрестках могут использоваться на перекрестках где движение или физические условия не оправдывают обычные сигналы управления дорожным движением, но частота аварий указывает на возможность особой потребности.

Контрольный маяк перекрестка обычно находится над центром перекрестка; тем не менее, он может использоваться в других подходящие места.

Раздел 4K.03 Предупреждение Маяк

Поддержка:
Типичные области применения предупреждающих маяков включают следующее:

  1. На препятствиях на проезжей части или в непосредственной близости от нее;
  2. В качестве дополнительного акцента на предупреждающих знаках;
  3. В качестве упора для пешеходных переходов в середине квартала;
  4. На подходах к перекресткам, где требуется дополнительное предупреждение, или при наличии особых условий; и
  5. В качестве дополнительного акцента к нормативным знакам, кроме СТОП, УХОД, НЕ ВХОДИТЕ и знаки ОГРАНИЧЕНИЯ СКОРОСТИ.

Стандарт:
Предупреждающий маяк должен состоять из одной или нескольких сигнальных секций стандартное лицо светофора с мигающим КРУГЛЫМ ЖЕЛТЫМ сигналом индикация в каждой секции сигнала.

Предупреждающий маяк должен использоваться только для дополнения соответствующий предупреждающий или регулирующий знак или маркер. Маяк не должны быть включены в границы знака, за исключением ШКОЛА Маяки знака ОГРАНИЧЕНИЕ СКОРОСТИ.

Предупреждающие маяки, если они используются на перекрестках, должны не сталкиваться с конфликтными автомобильными подходами.

Если проблесковый маячок подвешен над проезжей частью, просвет над тротуаром должен быть не менее 4,6 м (15 футов) но не более 5,8 м (19 футов).

Руководство:
Условия или правила, оправдывающие установку предупредительных маяков, должны в значительной степени определяют их расположение относительно проезжей части.

Если препятствие находится на проезжей части или рядом с ней, освещение нижней части или начала препятствия или знак на препятствии или перед ним, в дополнение к маяк, надо учитывать.

Предупреждающие маяки следует включать только во время те часы, когда существует условие или правило.

Опция:
Если предупреждающие маяки имеют более одной сигнальной секции, они могут мигают попеременно или одновременно.

Мигающий желтый маяк, связанный с движением Узел сигнального контроллера может использоваться с предупреждением о светофоре знак (см. раздел 2C.29).

Раздел 4K.04 Скорость Проблесковый маячок ограничительного знака

Стандарт:
Маяк знака ограничения скорости должен использоваться только в дополнение к знаку ограничения скорости. Предельный знак.

Маяк знака ограничения скорости должен состоять из одного или несколько сигнальных секций стандартного сигнального лица управления дорожным движением, с мигающим КРУГЛЫМ ЖЕЛТЫМ индикатором сигнала в каждом сигнале раздел. Сигнальные линзы должны иметь номинальный диаметр не менее 200 мм (8 дюймов). Если используются две линзы, они должны располагаться вертикально. выровнены, за исключением того, что они могут быть выровнены по горизонтали, если скорость Знак ограничения (R2-1) длиннее по горизонтали, чем по вертикали. Если два используются линзы, они должны попеременно мигать.

Опция:
Маяк знака ограничения скорости может использоваться с фиксированной или переменной скоростью. Предельный знак. Если применимо, мигающий маяк знака ограничения скорости (с соответствующий сопровождающий знак) может использоваться для обозначения того, что указанное ограничение скорости действует.

Поддержка:
Раздел 7B.11 содержит дополнительные опции для использования маяков ограничения скорости с Знаки ОГРАНИЧЕНИЯ СКОРОСТИ В ШКОЛЕ.

Секция 4K.

05 Стоп Маяк

Стандарт:
Стоп-маяк должен состоять из одной или нескольких сигнальных секций стандартное лицо светофора с мигающим КРУГЛЫМ КРАСНЫМ сигналом индикация в каждой секции сигнала. Если два горизонтально выровненных сигнала линзы, они должны мигать одновременно, чтобы избежать путают с сигнальными огнями на пересечении шоссе и железной дороги. Если используются две вертикально ориентированные сигнальные линзы, они должны быть мигали попеременно.

Нижняя часть корпуса сигнального стоп-сигнала должна быть не менее 300 мм (12 дюймов) и не более 600 мм (24 дюйма) над верхней частью знака СТОП (см. раздел 2Б.04).

Вернуться к началу

Сигналы

Светофоры устанавливаются на перекрестках для контроля движения транспортных средств и пешеходов. Сигналы светофора располагаются либо вертикальными, либо горизонтальными линиями. Когда они расположены вертикально, красный цвет всегда находится сверху, а зеленый — снизу. Когда они расположены горизонтально, красный всегда слева, а зеленый справа.

Красный, желтый и зеленый светофоры и стрелки

Когда есть ПОСТОЯННЫЙ КРАСНЫЙ СВЕТ , вы должны остановиться перед пересечением отмеченной стоп-линии или пешеходного перехода. Если вы не видите никаких линий, остановитесь перед въездом на перекресток. Прежде чем начать, дождитесь зеленого сигнала светофора.

Вы можете повернуть направо на красный свет, если только НЕ ПОВОРАЧИВАТЬ НА КРАСНЫЙ знак на перекрестке. Сначала вы должны остановиться, проверить и уступить дорогу пешеходам и другим транспортным средствам.

Вы также можете повернуть налево после остановки на красный свет, если вы находитесь в левой полосе и поворачиваете налево с улицы с односторонним движением на другую улицу с односторонним движением, если только знак не запрещает вам поворачивать. Сначала вы должны остановиться и уступить дорогу пешеходам и другим транспортным средствам.


А ПОСТОЯННЫЙ ЖЕЛТЫЙ СВЕТ говорит вам, что скоро появится постоянный красный свет. Если вы едете к перекрестку и загорается желтый сигнал светофора, снизьте скорость и приготовьтесь к остановке. Если вы находитесь в пределах перекрестка или не можете безопасно остановиться перед въездом на перекресток, осторожно продолжайте движение.


А ПОСТОЯННЫЙ ЗЕЛЕНЫЙ СВЕТ означает, что вы можете проехать через перекресток, если дорога свободна. Вы также можете повернуть направо или налево, если знак не говорит вам этого не делать; однако при повороте вы должны уступить дорогу другим транспортным средствам и пешеходам в пределах перекрестка.


А ПОСТОЯННАЯ КРАСНАЯ СТРЕЛКА означает, что вы должны остановиться и не можете поворачивать в направлении, указанном стрелкой. Прежде чем начать, дождитесь зеленой или мигающей желтой стрелки. Те же самые включения красного цвета, которые разрешены для постоянного красного сигнала, разрешены для постоянной красной стрелки.


А НЕПРЕРЫВНАЯ ЖЕЛТАЯ СТРЕЛКА означает, что движение, разрешенное зеленой стрелкой или мигающей желтой стрелкой, подходит к концу, и сигнал вскоре станет красным. Вы должны снизить скорость и подготовиться к полной остановке перед въездом на перекресток. Если вы находитесь в пределах перекрестка или не можете безопасно остановиться перед въездом на перекресток, вы можете с осторожностью завершить поворот, начатый по предыдущей стрелке.


А НЕПРЕРЫВНАЯ ЗЕЛЕНАЯ СТРЕЛКА означает, что вы можете повернуть, как указывает стрелка. Когда вы поворачиваете во время фазы зеленой стрелки, ваш поворот «защищен», потому что встречный транспорт останавливается красным светом. Если зеленая стрелка превращается в зеленый сигнал светофора, вы все равно можете повернуть в этом направлении, но сначала уступите дорогу пешеходам и встречному транспорту.


Мигающие сигналы


Мигающий красный свет имеет то же значение, что и СТОП знак. Вы должны полностью остановиться. Затем смотрите в обе стороны и продолжайте движение только после того, как перекресток будет свободен.


Мигающий желтый свет означает ОСТОРОЖНО . Притормози, посмотри и продолжай осторожно.


А МИГАЮЩАЯ ЖЕЛТАЯ СТРЕЛКА означает, что вы можете с осторожностью поворачивать в направлении стрелки, однако сначала вы должны уступить дорогу встречному транспорту и пешеходам. Когда вы поворачиваете во время фазы мигания желтой стрелки, ваш ход НЕ защищен; встречный транспорт будет иметь зеленый свет.


Неработающие светофоры


К неработающему светофору следует относиться как к четырехстороннему. СТОП знак.


Сигналы для измерения въезда


На некоторых въездах на шоссе в конце въезда установлены светофоры, которые попеременно горят зеленым и красным цветом. Сигналы рампы используются для контроля потока транспорта на очень загруженных автомагистралях и обычно работают в часы пик. Вы должны дождаться зеленого сигнала, прежде чем выехать на шоссе. На однополосных съездах только один автомобиль может выезжать на шоссе каждый раз, когда загорается зеленый свет. Для многополосных въездов на каждую полосу будет установлен счетчик пандуса. Зеленый свет не защищает ваш въезд на шоссе — вы все равно должны искать безопасную брешь в движении, чтобы выехать на шоссе.


Сигналы управления движением по полосам


Специальные сигналы также могут быть размещены непосредственно над полосами для управления движением. Они обычно используются в пунктах взимания платы, на мостах, туннелях и на многополосных дорогах в городах, где направление движения по определенной полосе меняется, чтобы облегчить въезд или выезд из города в час пик. Следующие сигналы показывают, как следует использовать определенные полосы движения на улице или шоссе:

Постоянная направленная вниз зеленая стрелка над полосой движения означает, что вы можете использовать эту полосу.


Постоянный желтый крестик над полосой движения означает, что вы должны перестроиться в другую полосу, потому что направление движения по этой полосе вот-вот изменится на противоположное. Приготовьтесь безопасно покинуть полосу движения.


Непрерывный красный крестик над полосой означает, что вам не разрешено использовать полосу движения.


Белая устойчивая стрелка одностороннего левого поворота над полосой движения означает, что вы можете повернуть налево, только если вы находитесь в этой полосе.


Белая устойчивая двусторонняя стрелка левого поворота над полосой означает, что вы можете повернуть налево только в том случае, если вы находитесь в этой полосе, но полосу делят водители, поворачивающие налево, приближающиеся с противоположного направления.


Сигналы для пешеходов


Пешеходы должны соблюдать дорожные и пешеходные сигналы. Сигналы пешеходов, которым вы должны подчиняться, ПРОГУЛКА и НЕ ХОДИТЕ огни или освещенное изображение идущего человека (имеется в виду ходить) и поднятой руки (имеется в виду не ходить). Если на перекрестке нет пешеходных сигналов, пешеходы должны подчиняться сигналам красного, желтого и зеленого сигналов светофора.

Иногда пешеходы не знают о своих обязанностях на светофорах и пешеходных сигналах. Будьте внимательны к пешеходам, которые могут не повиноваться сигналу. Всегда уступайте дорогу пешеходам.

Когда устойчивый ПРОХОДИТЕ или идет человек, начните переход, но будьте внимательны к поворачивающим транспортным средствам, которые не могут законно уступить дорогу.


Когда мигает НЕ ХОДИТЕ или мигание поднятой рукой начинается: a. Завершите переход, если вы находитесь на улице, b. Не начинайте переход, если вы не съехали с бордюра.


Когда устойчивый НЕ ХОДИТЕ отображается сообщение или устойчиво поднятая рука, не переходите дорогу.


Слепые пешеходы

Проезжая мимо слепого пешехода с белой тростью или с собакой-поводырем, необходимо снизить скорость, всегда уступать дорогу, а затем двигаться с осторожностью. Будьте готовы остановить автомобиль, чтобы предотвратить травму или опасность для пешехода.


Сигналы школьной зоны

Сигналы школьной зоны — это мигающие желтые сигналы, размещенные на знаках ограничения скорости в школьной зоне. Вы должны двигаться со скоростью не более 15 миль в час, когда мигают желтые сигналы или в периоды времени, указанные на знаке. Превышение скорости в школьной зоне приведет к трем баллам в вашем водительском стаже, а также к штрафу.


Сигналы железнодорожных переездов


Знак «Железнодорожный переезд» предупреждает водителей о приближении к железнодорожному переезду. Подходить к железнодорожным переездам следует всегда с осторожностью. Вы всегда должны смотреть в обе стороны и прислушиваться к приближающимся поездам или сигналам, прежде чем двигаться по железнодорожным путям.

Вы обязаны останавливаться на всех железнодорожных переездах, когда есть сигнал приближающегося поезда. Эти сигналы включают в себя мигающие красные огни, опущенные шлагбаумы, сигнализацию флагмана или предупреждающий звуковой сигнал поезда. Не двигайтесь вперед, не пытайтесь объехать какие-либо ворота и не игнорируйте сигнал приближающегося поезда. Если на железнодорожном переезде нет сигналов, вам следует снизить скорость и подготовиться к остановке, если вы увидите или услышите приближающийся поезд.

Двигайтесь с осторожностью только после того, как все сигналы будут завершены, и только тогда, когда есть доказательства того, что поезда не приближаются к переезду.

Вы должны остановиться, если поезд приближается и подал звуковой сигнал, или если он хорошо виден и находится в опасной близости от переезда.

Не останавливайте автомобиль на железнодорожных путях, даже если вам кажется, что приближается поезд. Если движение затруднено из-за светофора, знака «стоп» или по любой другой причине, убедитесь, что вы остановили свой автомобиль в месте, где он будет полностью свободен от каких-либо железнодорожных путей.

См. Глава 3 для получения дополнительной информации о том, как безопасно пересечь пересечение шоссе и железной дороги.


Нью-Йорк DMV | Глава 4: Управление движением

Темы:

  • Знаки
  • Традовые сигналы
  • Маркировка тротуара
  • Офицеры дорожного движения
  • ГЛАВА 4 ТОГ

ПРИМЕЧА правила дорожного движения (главы с 4 по 11 и дорожные знаки).

 

Знаки

Дорожные знаки сообщают вам о правилах дорожного движения, особых опасностях, где вы находитесь, как добраться туда, куда вы направляетесь, и где доступны услуги.

Форма и цвет дорожных знаков указывают на тип информации, которую они предоставляют:

НОРМАЛЬНЫЕ ЗНАКИ  обычно представляют собой белые прямоугольники с черными буквами или символами, но некоторые из них имеют другую форму, а некоторые могут использовать красные буквы или символы.

ПРЕДУПРЕЖДАЮЩИЕ ЗНАКИ  обычно желтого цвета в форме ромба с черными буквами или символами.

ЗНАКИ НАЗНАЧЕНИЯ зеленого цвета с белыми буквами и символами.

СЛУЖЕБНЫЕ ЗНАКИ  синего цвета с белыми буквами и символами.

Знайте показанные ниже знаки и их значение. Вас спросят о них во время письменного теста.


Вот описание общих трафик и то, что они указывают:

Пресс.

ЗНАЧЕНИЕ:  Полная остановка, уступить дорогу транспортным средствам и пешеходам, въезжающим на перекресток или направляющимся к нему. Иди, когда будет безопасно. Вы должны остановиться перед стоп-линией, если она есть. В противном случае вы должны остановиться перед переходом. (См. «Стоп-линии и линии пешеходного перехода» в разделе «Разметка тротуара» этой главы.)  Если стоп-линии или пешеходного перехода нет, вы должны остановиться перед въездом на перекресток в точке, ближайшей к перекрестку, откуда открывается вид на улицу. движение на пересекающейся проезжей части.

 

   Знак доходности        

ЦВЕТ: Красный и белый, с красными буквами.

ЗНАЧЕНИЕ:  Снижайте скорость по мере приближения к перекрестку. Приготовьтесь остановиться и уступить дорогу транспортным средствам и пешеходам, въезжающим на перекресток или направляющимся к нему. Вы должны полностью остановиться у знака YIELD, если этого требуют дорожные условия. Когда вы приближаетесь к знаку YIELD, внимательно проверьте движение и будьте готовы остановиться.

 

  Другое  Знаки правил       

ЦВЕТ:  Белый, с черными и/или красными буквами или символами.

ЗНАЧЕНИЕ:  Эти знаки информируют о правилах направления движения, использовании полосы движения, поворотах, скорости, парковке и других специальных требованиях.

Некоторые нормативные знаки имеют красный кружок с косой чертой над символом. Это указывает на то, что действие, такое как поворот направо, запрещено или что некоторым транспортным средствам запрещено движение по дороге. Прямоугольные белые знаки с черными или красными буквами или символами указывают на необходимость соблюдать особые правила.

 

ПРЕДУПРЕЖДАЮЩИЕ ЗНАКИ:    

    

ЦВЕТ:  – желтый, с черными буквами или желтым.

ЗНАЧЕНИЕ:  Вы приближаетесь к опасному месту или месту, где действуют особые правила, как показано на образцах знаков. Иногда к предупреждающему знаку присоединяется желто-черный знак «рекомендуемая скорость». Это указывает на то, что в этом районе рекомендуется снизить скорость.

   

 

  Знаки рабочей зоны      

ЦВЕТ:  Оранжевый, с черными буквами или символами.

ЗНАЧЕНИЕ:  Люди работают на проезжей части или рядом с ней, и движение может контролироваться сигнальщиком. Ограничение скорости в рабочей зоне может быть установлено на уровне 25 миль в час (40 км/ч). Даже если ограничение скорости не предусмотрено, вы должны двигаться на пониженной скорости через рабочую зону и всегда должны подчиняться флагманам. На этих иллюстрациях показаны некоторые сигналы, которые будет использовать флагман. Знайте и соблюдайте их.


Стоп

Продолжайте

медленное

Знаки назначения

: Знаки пункта назначения

: .

ЗНАЧЕНИЕ: Показать направление и расстояние до мест.

 

Маршрутные знаки     

ЦВЕТ:  Разные.

ЗНАЧЕНИЕ:  Указывает маршруты между штатами, США, штатами или округами. Форма сообщает вам тип маршрута, на котором вы находитесь. Образцы знаков слева направо предназначены для маршрутов штатов, США и между штатами. Когда вы планируете поездку, используйте карту автомобильных дорог, чтобы решить, какие маршруты выбрать. Во время поездки следите за указателями пункта назначения, чтобы не заблудиться, не свернуть или резко остановиться.

 

Сервисные знаки  

ЦВЕТ: Синий, с белыми буквами или символами.

ЗНАЧЕНИЕ: Показывает расположение служб, таких как зоны отдыха, заправочные станции, кемпинги и медицинские учреждения.

 

Сигналы светофора  

  

Светофоры

Светофоры обычно имеют красный, желтый и зеленый цвета сверху вниз или слева направо. На некоторых перекрестках есть одиночные красные, желтые или зеленые огни. Одни светофоры горят постоянно, другие мигают. Некоторые из них круглые, а некоторые — стрелки. Закон штата требует, чтобы, если светофор или средства управления вышли из строя или не работают должным образом, когда вы приближаетесь к перекрестку, вы должны остановиться, как если бы вы остановились на знаке «Стоп». Затем вы должны продолжать движение в соответствии с правилами преимущественного проезда, если только вам не велел продолжать движение сотрудник службы дорожного движения.


Вот что показывают разные светофоры:

Горит красным:  Стоп. Не уходите, пока не загорится зеленый свет. Если показана зеленая стрелка с красным светом, вы можете двигаться в направлении стрелки и только в том случае, если перекресток свободен.

Вы можете повернуть направо на красный сигнал светофора после полной остановки и уступить дорогу встречному транспорту и пешеходам. Вы можете повернуть налево при постоянном красном свете, когда поворачиваете с дороги с односторонним движением на другую дорогу с односторонним движением после полной остановки и уступаете право проезда встречному транспорту и пешеходам.

Вы не можете повернуть на красный свет, если вывешен знак НЕ ПОВОРАЧИВАТЬ НА КРАСНЫЙ или другой знак, сигнал или разметка тротуара препятствуют повороту. Вам не разрешается включать красный свет в Нью-Йорке, если не вывешен знак, разрешающий это.

Водитель школьного автобуса с учениками не может повернуть направо на красный свет.

 

 Мигающий красный: Означает то же, что и знак СТОП: остановитесь, уступите дорогу и идите, когда это будет безопасно.

 

R ed Стрелка: Не двигайтесь в направлении, указанном стрелкой, пока не погаснет красная стрелка и не загорится зеленый свет или стрелка. На красной стрелке поворот направо или налево на красный запрещен.

Постоянный желтый: Цвет индикатора изменится с зеленого на красный. Будьте готовы остановиться на красный свет.

 Желтый мигающий: Будьте осторожны при вождении.
 

 

Желтая стрелка: Защита зеленой стрелки закончится. Если вы собираетесь повернуть в направлении стрелки, будьте готовы остановиться.

 

Постоянно горит зеленым: Продолжайте движение, но уступайте право проезда другим транспортным средствам на перекрестке, как того требует закон (см. главу 5).

 

 Зеленая стрелка: Вы можете двигаться в направлении, указанном стрелкой, но вы должны уступить дорогу другим транспортным средствам на перекрестке в соответствии с требованиями закона (см. Главу 5.)  

 

Контрольные лампы использования полосы движения  

Специальные лампы над тротуаром иногда используются для обозначения полос движения, которые можно использовать в определенное время:

Горит красным «X»:  Не двигайтесь по этой полосе.

Немигающий желтый крестик:  Двигайтесь с этой полосы.

Мигающий желтый «X»:  Эту полосу можно использовать только для левого поворота.

Зеленая стрелка:  Вы можете использовать эту полосу.

 

Дорожная разметка

Линии и символы на проезжей части разделяют полосы и сообщают вам, когда вы можете обгонять другие транспортные средства или перестраиваться. Они также сообщают вам, какие полосы использовать для поворотов и где вы должны остановиться для знаков или сигналов светофора. Стрелки на этих рисунках показывают направление движения.

Линии обочины и полосы движения

Сплошные линии вдоль обочины указывают, где находится ее край – где заканчивается полоса движения и начинается обочина. Запрещается переезжать границу, за исключением случаев, когда об этом говорит сотрудник полиции или другое уполномоченное должностное лицо, или когда это разрешено официальным знаком. Краевая линия, направленная под углом к ​​центру дороги, показывает, что дорога впереди уже. Линии, разделяющие полосы движения в одном направлении, окрашены в белый цвет.

Линии, разделяющие транспортные средства, движущиеся в противоположных направлениях, выделены желтым цветом. Между полосами движения могут быть две линии, а линии могут быть сплошными или прерывистыми. Прочтите Главу 6 , чтобы узнать о правилах обгона других транспортных средств.

 

Что обозначают некоторые полосы движения:

Одна   прерывистая линия: Вы можете обгонять другие транспортные средства или менять полосу движения, если можете сделать это безопасно, не мешая движению транспорта.

Сплошная прерывистая линия:  Если вы находитесь на стороне со сплошной линией, вы не можете обгонять другие транспортные средства или пересекать линию, кроме как повернуть налево на подъездную дорожку. Если вы находитесь на стороне с прерывистой линией, вы можете проехать, если это безопасно и вы не будете мешать движению.

Двойные сплошные линии: Вы не можете обгонять или перестраиваться. Вы не можете пересекать линии, кроме как повернуть налево, чтобы въехать на шоссе или съехать с него (например, въехать на подъездную дорожку или свернуть с нее или сделать разворот, см. главу 5).

  

Одна сплошная линия: вы можете объезжать другие транспортные средства или менять полосу движения, но вы можете делать это только в том случае, если этого требуют препятствия на дороге или дорожные условия.

Линии остановки и пешеходного перехода:  На перекрестке, контролируемом знаком «СТОП», знаком «УЙДИТЕ» или светофором, через полосу движения может быть нанесена белая стоп-линия (называемая стоп-линией) и/или две параллельные линии. нарисовано через дорогу (так называемый пешеходный переход). Когда требуется остановиться из-за знака или светофора, вы должны остановиться, не дойдя до стоп-линии, если она есть, или до пешеходного перехода. Вам нужно останавливаться на стоп-линии или пешеходном переходе только в том случае, если этого требуют светофоры, знаки или сотрудники службы дорожного движения, или уступать дорогу пешеходу, роликовому конькобежцу или самокату на отмеченном или немаркированном пешеходном переходе. Единая стоп-линия может быть размещена на перекрестках, чтобы дать возможность более крупным транспортным средствам (например, тягачам с прицепами, автобусам и грузовикам) поворачивать, не заставляя другой транспорт двигаться задним ходом. Важно остановиться до того, как вы достигнете этой стоп-линии. (См. «Пешеходы» в главе 11).

   

Стрелки: Стрелки показывают, какие полосы вы должны использовать. На этой иллюстрации, например, вы можете повернуть направо только с правой полосы. Чтобы ехать прямо, вы должны использовать левый ряд. Вы должны находиться в правильной полосе, прежде чем достигнете сплошной линии, разделяющей полосы.


Ромбовидный символ:
  Этот символ указывает на зарезервированные полосы для автобусов, HOV (автомобилей с высокой вместимостью), таких как автомобильные парки и фургоны, велосипеды или другие специальные транспортные средства. Вы не можете въезжать на эти полосы и использовать их, если ваш автомобиль не соответствует требованиям по вместимости или другим требованиям, указанным знаками в период действия особых условий. При использовании для обозначения зарезервированных полос на городских улицах участки сплошной белой линии, отделяющей ромбовидные полосы от обычных полос, можно заменить прерывистыми белыми линиями. В этих местах автомобили без HOV могут выехать на полосу движения для HOV, если они повернут направо на следующем перекрестке. Автобусные полосы и полосы движения HOV должны способствовать наиболее эффективному использованию ограниченной пропускной способности улиц и автомагистралей. Они гарантируют, что наиболее важные транспортные средства будут двигаться быстрее всех.

Инспекторы дорожного движения

Указания, данные инспекторами дорожного движения, имеют приоритет над знаками, сигналами или дорожной разметкой. Например, если дорожный инспектор сигнализирует вам остановиться на зеленый свет, вы должны остановиться. Если офицер сигнализирует вам проехать на красный свет или знак остановки, вы должны это сделать.

Среди лиц, уполномоченных управлять дорожным движением, есть сотрудники полиции, пожарной полиции, сигнальщики на дорожных работах, лица, пересекающие школьный переход, и водители школьных автобусов.

 


Вопросы

Прежде чем перейти к главе 5, убедитесь, что вы можете определить знаки в этой главе и понять, что они означают. Кроме того, убедитесь, что вы можете ответить на следующие вопросы:

  • Какой формы обычно бывает нормативный знак?
  • Каков нормальный цвет и форма предупреждающего знака?
  • Какого цвета и формы знак назначения?
  • Что нужно делать при знаке СТОП?
  • Что вы должны делать, если столкнулись с каждым из следующих явлений: мигающим красным светом, мигающим желтым светом, постоянным желтым светом, красным светом с зеленой стрелкой?
  • На что указывает наклон краевой линии к центру дороги?
  • Что обозначает каждая из этих линий: одна прерывистая, одна сплошная, двойная сплошная, сплошная и разорванная вместе?
  • Если на перекрестке есть линии пешеходного перехода, но нет линии СТОП, где вы должны остановиться на красный свет на этом перекрестке?
  • Какой тип дорожной разметки используется, чтобы показать вам, какую полосу вы должны использовать для поворота?
  • Что из следующего вы должны подчиняться трем другим: постоянному красному свету, мигающему красному свету, знаку СТОП, полицейскому?

 

  Конец главы 4:  Викторина по главе 4

< Глава 3  | Содержание  |   Глава 5 >  

 

Зеленая вспышка: легенды, предания и наука о феномене заката — Фермерский альманах

Зеленая вспышка достигла мифического статуса из-за своей редкости и непредсказуемости.

Узнайте, как и когда можно увидеть это неуловимое оптическое явление и чем оно вызвано.

Натали ЛаВольп Обновлено: 7 сентября 2022 г.

Не путать с вымышленным супергероем «Зеленой вспышкой». Солнечная «зеленая вспышка» — это столь же неуловимое природное явление. На протяжении веков мореплаватели рассказывали многочисленные зловещие истории об этих волшебных вспышках зеленого света прямо над Солнцем, когда они наблюдали, как оно садится на воду. Но скептики отмахнулись от их историй, заявив, что это миф. В то время как большинство пиратских преданий можно отнести к фантастической фантастике, зеленая вспышка таковой не является. Эта едва заметная иллюзия — настоящее зрелище, подкрепленное наукой. Так что же это такое, и как вы можете это увидеть?

Что такое зеленая вспышка?

Зеленая вспышка — это явление, при котором часть Солнца внезапно становится ярко-зеленой примерно на 1–2 секунды. Короткая вспышка зеленого света чаще видна на закате, чем на восходе.

Что вызывает зеленую вспышку?

Неуловимую зеленую вспышку можно увидеть и над землей, когда хорошо виден горизонт.

Так что же вызывает эту вспышку цвета? Атмосфера Земли действует как призма, разделяя солнечный свет на разные цвета. Зеленая вспышка — результат наблюдения лучей через все более плотную атмосферу. Когда Солнце восходит или опускается за горизонт, свет преломляется и рассеивается в атмосфере. Различные цвета изгибаются в разной степени в зависимости от длины волны. Цвета с более короткими длинами волн, такие как синий, зеленый и фиолетовый, отражают сильнее, чем цвета с более длинными волнами, такие как красный, оранжевый и желтый.

Когда Солнце находится выше в небе, свет проходит меньшее расстояние, и цвета плохо различаются. Однако, поскольку Солнце опускается за горизонт, свет должен проходить дальше через атмосферу, чтобы встретиться с вашими глазами. В результате цвета легче разделяются. Когда условия идеально подходят, волны зеленого цвета достигают наших глаз, в то время как другие цвета отфильтровываются. Вот почему мы иногда можем видеть эту зеленую вспышку света.

Типы зеленых вспышек

Не все зеленые вспышки одинаковы. На самом деле их несколько типов:

  • Нижний мираж: Эти более распространенные вспышки имеют овальную и плоскую форму и возникают близко к уровню моря, когда поверхность воды теплее воздуха.
  • Мнимый мираж: Эти вспышки появляются тонкими заостренными полосами, длящимися 1-2 секунды, и происходят выше в небе, когда поверхность воды холоднее, чем воздух над ней.
  • Субдуктовые вспышки: Эта реже встречающаяся вспышка возникает из-за атмосферной инверсии (слой теплого воздуха задерживает холодную воздушную влагу близко к земле). Кажется, что Солнце имеет форму песочных часов, а верхняя часть Солнца кажется зеленой в течение более 15 секунд.
  • Зеленый луч: Самый редкий тип зеленой вспышки возникает, когда луч зеленого света вырывается прямо из зеленой вспышки сразу после захода Солнца. Это вызвано сочетанием дымки в воздухе и одного из трех других типов вспышек.

Откуда взялось это название?

Имя зеленой вспышки можно отнести к роману французского писателя Жюля Верна 1882 года « Le Rayon-Vert » («Зеленый луч»), в котором героиня ищет таинственное явление однозначного очарования. Верн описывает этот цвет как «зеленый, которого ни один художник никогда не мог получить на своей палитре».

Зеленая вспышка Знания

Говорят, что однажды увидев зеленую вспышку, вы больше никогда не ошибетесь в сердечных делах. Но если вы поклонник фильмов «Пираты Карибского моря», вы знаете, что зеленая вспышка означает «душу, [которая] возвращается в этот мир из мертвых».

Посмотрите клип на отметке 1:46: здесь упоминается зеленая вспышка.

Флэш также помогает в изучении погоды. Старая английская пословица утверждает:

Взгляни на зеленый луч,
Считай завтрашний день прекрасным.

Этот отрывок из предания о погоде может напомнить вам пословицу «Красное небо ночью», в которой цвет неба на закате предсказывает погоду на следующий день. В этом случае зеленая вспышка на закате означает, что наблюдатель, обращенный на запад, смотрит на ясный горизонт, откуда придет завтрашняя погода (из-за направления струйного течения), поэтому погода на следующий день будет «хорошей».

Как увидеть зеленую вспышку

Перспектива наблюдать за неуловимой вспышкой стала для многих квестом. Если вы окажетесь в оптимальном месте, в идеальных условиях, возможно, вам просто повезло наблюдать за этим оптическим явлением. Зеленую вспышку чаще всего можно увидеть на закате, но ее можно поймать и на восходе. Однако наблюдать за восходом Солнца сложнее, так как ваши глаза должны быть сфокусированы точно на том месте на горизонте, где появится Солнце, что сложно сделать.

Зеленая вспышка достигла мифического статуса из-за своей редкости и непредсказуемости. В отличие от других небесных явлений, таких как метеоритные дожди или кометы, когда ученые могут заранее сообщить об их появлении, невозможно предсказать, когда вы сможете мельком увидеть зеленую вспышку. Несмотря на его спонтанный и неуловимый характер, есть способы увеличить ваши шансы поймать это мимолетное световое шоу.

В нормальных условиях зеленые вспышки настолько тонкие и быстрые, а интенсивность «вспышек» обычно слишком слаба, чтобы их можно было уловить человеческим глазом. Однако, если атмосферные условия идеальны, эффект может быть достаточно усилен, чтобы его можно было наблюдать. Лучший способ обеспечить успешное наблюдение за зеленой вспышкой — убедиться, что у вас есть следующее:

  • Ясный день без дымки или облаков
  • Беспрепятственный обзор горизонта с отчетливым краем

Зеленые вспышки обычно видны над океаном, где видна большая часть атмосферы, а линия обзора параллельна горизонт. Но если ваш горизонт достаточно далеко, их можно увидеть и на суше. Сидение на более высокой точке также может повысить ваши шансы мельком увидеть. Говорят, что зеленые вспышки регулярно видны с вершины Эмпайр-стейт-билдинг, с вершин гор, а также с самолетов, летящих на запад, и на закате.

Важно помнить, что нельзя смотреть прямо на Солнце, пока оно не окажется почти прямо за горизонтом, иначе можно повредить глаза.

Захват зеленой вспышки на камеру

К счастью, камеры лучше способны обнаруживать слабые вспышки, чем человеческий глаз. Чтобы увеличить свои шансы заснять свою собственную зеленую вспышку, настройте камеру непосредственно перед закатом вечером, когда горизонт чист и свободен от облаков и дымки. Чтобы поймать его на рассвете, нужно больше гадать о том, где и когда взойдет солнце, поэтому на закате вам может повезти больше.

Расскажите нам: вы когда-нибудь видели зеленую вспышку? Поделитесь своей историей в комментариях ниже.

Натали ЛаВольп

Натали ЛаВольп — писатель-фрилансер и бывший учитель специального образования. Она посвящена здоровому образу жизни через тело и разум. В настоящее время она проживает на Лонг-Айленде, штат Нью-Йорк, с мужем, детьми и собакой.

управление воздушным движением — Что на самом деле означает мигающий зеленый сигнал (возврат на посадку) для самолетов NORDO?

спросил

Изменено 2 года, 6 месяцев назад

Просмотрено 3к раз

$\begingroup$

Изучение для PSTAR Я вижу, что мигающий зеленый сигнал на схеме означает «возврат на посадку», но это нигде не определено. Что это значит?

NORDO = «самолёт без радио»

управление воздушным движением посадка схема движения канада аэропорт-освещение

$\endgroup$

$\begingroup$

Меня учили, что это означает…

… потому что они не могли связаться с вами, но хотели, чтобы вы не приземлялись на этом перевале (интервал — бардак, перекресток, ветер может быть неправильный, а дюжина других вещей, которые могут возникнуть ) это было более-менее «ВПП будет готова для вас, если вы сделаете еще один круг по схеме». Это также дает им немного времени, чтобы убрать все остальные самолеты в схеме с пути терпящего бедствие самолета. (И постоянный зеленый цвет должен подтвердить это на следующем проходе, если все пойдет хорошо).

Операторы башни предпочли бы, чтобы тот самолет, который не может с ними разговаривать, сел на землю. Чего они хотят избежать в этом случае (опять же, это то, чему меня научил старый соленый диспетчер ВМФ, когда я учился в летной школе), так это того, что они хотели избежать того, чтобы самолет, пытающийся приземлиться, был «напуган» красным светом и думая, что они не могут приземлиться на этом поле. А потом отмахнуться и улететь на поиски другого поля еще НОРДО.

Зеленый свет — это «дружественный» свет между красным и зеленым. На разговорном жаргоне командира: «Мигающий зеленый сигнализирует пилоту, что «взлетно-посадочная полоса будет там, когда вы вернетесь для следующего захода на посадку». которые имеют отношение к этому вопросу, можно найти здесь

Обучение, которое я получил, возможно, больше не актуально, но огни, показанные бортовому самолету NORDO, по-прежнему означают то же самое, что и когда меня учили 9.0003

Немигающий зеленый: разрешение на посадку
Мигающий зеленый: возвращение на посадку (в нужное время за ним загорится немигающий зеленый)

$\endgroup$

3

$\begingroup$

Понимание сигналов световой пушки меня учили немного по-другому. Для простоты я сосредоточусь на сигналах световых орудий только для бортовых самолетов.

  • Чередование красного и зеленого требует особой осторожности.
  • Постоянный красный свет уступает дорогу другому транспорту. Это означает, что вы должны оставаться в шаблоне. Но вам не разрешено приземляться.
  • Steady Green — разрешение на посадку.
  • Мигающий красный — просьба покинуть территорию аэропорта. Это будет означать, что вам нужно оставаться вне схемы движения.
  • Мигающий зеленый сигнал — рассматриваемый сигнал. Основываясь на определениях других сигналов, это означает, что вы должны войти в модель или вернуться к ней. Но вы по-прежнему не можете приземлиться, пока не получите устойчивый зеленый цвет.

В двух словах, чтобы было легче запомнить:

  • Зеленый означает одобрение, а красный означает отказ.
  • Устойчивые средства для взлетно-посадочной полосы и Мигающие средства для шаблона.

$\endgroup$

2

$\begingroup$

Приведенные выше ответы в значительной степени верны, но ссылки на источники не помешают.

Поскольку вопрос касается обучения на PSTAR и помечен как Канада, сначала канадские источники:

Канада

Руководство по авиационной информации (Канада)

Раздел 4.2.11 Визуальные сигналы — Самолет на земле

4.4.7 Визуальные сигналы — Самолет в полете

Соединенные Штаты

Канада. сигналы аналогичны, но не совсем такие же, как сигналы УВД США FAA, поэтому добавьте их для пилотов США, которые найдут этот вопрос

§ 91.125 Световые сигналы УВД

описано в Разделе 4-3-13

Различия

  • Канада не определяет использование чередующегося красного/зеленого сигнала
  • США не указывает ни использование мигающих огней взлетно-посадочной полосы, ни включение красных пиротехнических огней
  • .

$\endgroup$

1

$\begingroup$

По логике, это означает, что нужно лететь по кругу вокруг схемы движения и ждать постоянного зеленого сигнала светофора на следующем заходе на посадку.

Leave a Comment

Ваш адрес email не будет опубликован. Обязательные поля помечены *